Line 17: Line 17:
 
<math>\color{blue}\text{Solution 1:}</math>  
 
<math>\color{blue}\text{Solution 1:}</math>  
  
 +
<math>\left.\begin{matrix}
 +
x\left ( 1 \right )=2x\left ( 0 \right )+\mu\left ( 0\right )\\
 +
x\left ( 2 \right )=2x\left ( 1 \right )+\mu\left ( 1\right )\\
 +
x\left ( 3 \right )=2x\left ( 2 \right )+\mu\left ( 2\right )\\
 +
x\left ( 0 \right )=0
 +
\end{matrix}\right\} \Rightarrow
 +
\left\{\begin{matrix}
 +
x\left ( 1 \right )=\mu\left ( 0 \right )\\
 +
x\left ( 2 \right )=2\mu\left ( 0 \right )+\mu\left ( 1\right )\\
 +
x\left ( 3 \right )=4\mu\left ( 0 \right )+2\mu\left ( 1\right )+\mu\left ( 2 \right )=7
 +
\end{matrix}\right.</math><br>
  
 +
<math>\text{The problem is equivalent to minimize }</math>
  
 
+
<br>
  
 
----
 
----
  
<math>\color{blue}\text{Solution 2:}</math>
+
<math>\color{blue}\text{Solution 2:}</math>  
  
 
<math>x\left ( 1 \right )=\mu\left ( 0 \right )</math><br>  
 
<math>x\left ( 1 \right )=\mu\left ( 0 \right )</math><br>  
  
<math>x\left ( 2 \right )=2\mu\left ( 0 \right )+\mu\left ( 1 \right )</math>
+
<math>x\left ( 2 \right )=2\mu\left ( 0 \right )+\mu\left ( 1 \right )</math>  
  
<math>x\left ( 3 \right )=4\mu\left ( 0 \right )+2\mu\left ( 1\right )+\mu\left ( 2 \right )=7</math>
+
<math>x\left ( 3 \right )=4\mu\left ( 0 \right )+2\mu\left ( 1\right )+\mu\left ( 2 \right )=7</math>  
  
<math>\text{The problem transfer to min } J\left ( \mu  \right )=\frac{1}{2} \mu \left ( 0 \right )^{2}+\frac{1}{2} \mu \left ( 1 \right )^{2}+\frac{1}{2} \mu \left ( 2 \right )^{2}</math>
+
<math>\text{The problem transfer to min } J\left ( \mu  \right )=\frac{1}{2} \mu \left ( 0 \right )^{2}+\frac{1}{2} \mu \left ( 1 \right )^{2}+\frac{1}{2} \mu \left ( 2 \right )^{2}</math>  
  
<math>\text{subject to } h(\mu )=4\mu \left(0 \right)+2\mu \left(1 \right)+\mu\left(2 \right)-7=0</math>
+
<math>\text{subject to } h(\mu )=4\mu \left(0 \right)+2\mu \left(1 \right)+\mu\left(2 \right)-7=0</math>  
  
<math>\text{Apply KKT condition: } Dl\left( \mu ,\lambda \right)=DJ\left(\mu  \right)+\lambda Dh\left(\mu \right)=\left[ \mu\left(0  \right)+4\lambda,\mu\left(1  \right)+2\lambda,\mu\left(2  \right)+\lambda \right]=0</math>
+
<math>\text{Apply KKT condition: } Dl\left( \mu ,\lambda \right)=DJ\left(\mu  \right)+\lambda Dh\left(\mu \right)=\left[ \mu\left(0  \right)+4\lambda,\mu\left(1  \right)+2\lambda,\mu\left(2  \right)+\lambda \right]=0</math>  
  
 
<math>\left\{\begin{matrix}
 
<math>\left\{\begin{matrix}
Line 49: Line 61:
 
\mu\left(2 \right)=\frac{1}{3}\\  
 
\mu\left(2 \right)=\frac{1}{3}\\  
 
\lambda=-\frac{1}{3}
 
\lambda=-\frac{1}{3}
\end{matrix}\right.</math>
+
\end{matrix}\right.</math>  
  
 
<math>\text{Check SOSC: } L\left( \mu,\lambda  \right)=D^{2}l\left( \mu,\lambda \right)=\begin{bmatrix}
 
<math>\text{Check SOSC: } L\left( \mu,\lambda  \right)=D^{2}l\left( \mu,\lambda \right)=\begin{bmatrix}
Line 55: Line 67:
 
0 & 1 & 0\\  
 
0 & 1 & 0\\  
 
0 & 0 & 1
 
0 & 0 & 1
\end{bmatrix}>0</math>
+
\end{bmatrix}>0</math>  
  
<math>\therefore \text{For all y, } y^{T}Ly\geq 0</math>
+
<math>\therefore \text{For all y, } y^{T}Ly\geq 0</math>  
  
<math>\therefore \text{sequence } \left\{ \frac{4}{3},\frac{2}{3},\frac{1}{3} \right\} \text{ satisfy SOSC is a strict minimizer of the problem.}</math>
+
<math>\therefore \text{sequence } \left\{ \frac{4}{3},\frac{2}{3},\frac{1}{3} \right\} \text{ satisfy SOSC is a strict minimizer of the problem.}</math>  
  
 
<br>  
 
<br>  

Revision as of 18:17, 27 June 2012


ECE Ph.D. Qualifying Exam: Automatic Control (AC)- Question 3, August 2011


 $ \color{blue}\text{4. } \left( \text{20 pts} \right) \text{ Consider the following model of a discrete-time system, } $

$ x\left ( k+1 \right )=2x\left ( k \right )+u\left ( k \right ), x\left ( 0 \right )=0, 0\leq k\leq 2 $

$ \color{blue}\text{Use the Lagrange multiplier approach to calculate the optimal control sequence} $

$ \left \{ u\left ( 0 \right ),u\left ( 1 \right ), u\left ( 2 \right ) \right \} $

$ \color{blue}\text{that transfers the initial state } x\left( 0 \right) \text{ to } x\left( 3 \right)=7 \text{ while minimizing the performance index} $
$ J=\frac{1}{2}\sum\limits_{k=0}^2 u\left ( k \right )^{2} $

$ \color{blue}\text{Solution 1:} $

$ \left.\begin{matrix} x\left ( 1 \right )=2x\left ( 0 \right )+\mu\left ( 0\right )\\ x\left ( 2 \right )=2x\left ( 1 \right )+\mu\left ( 1\right )\\ x\left ( 3 \right )=2x\left ( 2 \right )+\mu\left ( 2\right )\\ x\left ( 0 \right )=0 \end{matrix}\right\} \Rightarrow \left\{\begin{matrix} x\left ( 1 \right )=\mu\left ( 0 \right )\\ x\left ( 2 \right )=2\mu\left ( 0 \right )+\mu\left ( 1\right )\\ x\left ( 3 \right )=4\mu\left ( 0 \right )+2\mu\left ( 1\right )+\mu\left ( 2 \right )=7 \end{matrix}\right. $

$ \text{The problem is equivalent to minimize } $



$ \color{blue}\text{Solution 2:} $

$ x\left ( 1 \right )=\mu\left ( 0 \right ) $

$ x\left ( 2 \right )=2\mu\left ( 0 \right )+\mu\left ( 1 \right ) $

$ x\left ( 3 \right )=4\mu\left ( 0 \right )+2\mu\left ( 1\right )+\mu\left ( 2 \right )=7 $

$ \text{The problem transfer to min } J\left ( \mu \right )=\frac{1}{2} \mu \left ( 0 \right )^{2}+\frac{1}{2} \mu \left ( 1 \right )^{2}+\frac{1}{2} \mu \left ( 2 \right )^{2} $

$ \text{subject to } h(\mu )=4\mu \left(0 \right)+2\mu \left(1 \right)+\mu\left(2 \right)-7=0 $

$ \text{Apply KKT condition: } Dl\left( \mu ,\lambda \right)=DJ\left(\mu \right)+\lambda Dh\left(\mu \right)=\left[ \mu\left(0 \right)+4\lambda,\mu\left(1 \right)+2\lambda,\mu\left(2 \right)+\lambda \right]=0 $

$ \left\{\begin{matrix} \mu\left(0 \right)+4\lambda=0\\ \mu\left(1 \right)+2\lambda=0\\ \mu\left(2 \right)+\lambda=0\\ 4\mu\left(0 \right)+2\mu\left(1 \right)+\mu\left(2 \right)-7=0 \end{matrix}\right. \Rightarrow \left\{\begin{matrix} \mu\left(0 \right)=\frac{4}{3}\\ \mu\left(1 \right)=\frac{2}{3}\\ \mu\left(2 \right)=\frac{1}{3}\\ \lambda=-\frac{1}{3} \end{matrix}\right. $

$ \text{Check SOSC: } L\left( \mu,\lambda \right)=D^{2}l\left( \mu,\lambda \right)=\begin{bmatrix} 1 & 0 & 0\\ 0 & 1 & 0\\ 0 & 0 & 1 \end{bmatrix}>0 $

$ \therefore \text{For all y, } y^{T}Ly\geq 0 $

$ \therefore \text{sequence } \left\{ \frac{4}{3},\frac{2}{3},\frac{1}{3} \right\} \text{ satisfy SOSC is a strict minimizer of the problem.} $



Back to ECE Qualifying Exams (QE) page

Alumni Liaison

Ph.D. on Applied Mathematics in Aug 2007. Involved on applications of image super-resolution to electron microscopy

Francisco Blanco-Silva